Re: [obm-l] Re: [obm-l] n^n = n (mod 8) para n ímpar

2018-03-26 Por tôpico Claudio Buffara
n^2 == 1 (mod 8) se n é ímpar.
Pra ver isso, basta testar n = 1, 3, 5, 7.
Daí e’ só elevar ambos os lados da congruência ao expoente (n-1)/2, obtendo:
n^(n-1) == 1 (mod 8).
Finalmente, multiplique esta congruência por n.

Abs

Enviado do meu iPhone

Em 26 de mar de 2018, à(s) 22:22, Anderson Torres 
 escreveu:

> Em 25 de março de 2018 15:28, Artur Steiner
>  escreveu:
>> Embora simples, acho interessante mostrar isso (aqui, = significa congruente
>> a). Parece não ser muito conhecido.
>> 
>> Artur Costa Steiner
>> 
> 
> Binômio de Newton?
> 
> Se n=2k+1 com k inteiro, temos (2k+1)^n = soma{0 <= j <= n} binom(n,j) (2k)^j
> 
> Módulo 8, só precisamos olhar j=0,1,2:
> 
> binom(n,2)4k^2 + binom(n,1)2k + binom(n,0)
> 
> n(n-1) * 2k^2+n * 2k + 1
> 
> (2k+1)*2k * 2k^2+(2k+1) * 2k + 1
> 
> 4k^3*(2k+1) +(2k+1) * 2k + 1
> 
> 8k^4* + 4k^3 +(2k+1) * 2k + 1
> 
> 4k^3 + 4k^2+ 2k + 1
> 
> 4k^2(k+1) +2k+1
> 
> É claro que k(k+1) é par, logo 4k^2(k+1) é 0 módulo 8
> 
> E isso nos deixa com 2k+1=n
> 
> 
>> --
>> Esta mensagem foi verificada pelo sistema de antivírus e
>> acredita-se estar livre de perigo.
> 
> -- 
> Esta mensagem foi verificada pelo sistema de antivírus e
> acredita-se estar livre de perigo.
> 
> 
> =
> Instruções para entrar na lista, sair da lista e usar a lista em
> http://www.mat.puc-rio.br/~obmlistas/obm-l.html
> =

-- 
Esta mensagem foi verificada pelo sistema de antiv�rus e
 acredita-se estar livre de perigo.


=
Instru��es para entrar na lista, sair da lista e usar a lista em
http://www.mat.puc-rio.br/~obmlistas/obm-l.html
=


[obm-l] Re: [obm-l] Revista para olímpicos (gratuita, online)

2018-03-26 Por tôpico Anderson Torres
Gostei! Vou até enviar...

Em 5 de fevereiro de 2018 10:44, Tássio Naia  escreveu:
> Salve,
>
> Gostaria de sugerir aos colegas a leitura do Archimede Mathematical Journal,
> um periódico voltado para olímpicos.
>
> http://amj-math.com/
>
> Até,
> Tássio
>
> --
> Esta mensagem foi verificada pelo sistema de antivírus e
> acredita-se estar livre de perigo.

-- 
Esta mensagem foi verificada pelo sistema de antiv�rus e
 acredita-se estar livre de perigo.


=
Instru��es para entrar na lista, sair da lista e usar a lista em
http://www.mat.puc-rio.br/~obmlistas/obm-l.html
=


[obm-l] Re: [obm-l] n^n = n (mod 8) para n ímpar

2018-03-26 Por tôpico Anderson Torres
Em 25 de março de 2018 15:28, Artur Steiner
 escreveu:
> Embora simples, acho interessante mostrar isso (aqui, = significa congruente
> a). Parece não ser muito conhecido.
>
> Artur Costa Steiner
>

Binômio de Newton?

Se n=2k+1 com k inteiro, temos (2k+1)^n = soma{0 <= j <= n} binom(n,j) (2k)^j

Módulo 8, só precisamos olhar j=0,1,2:

binom(n,2)4k^2 + binom(n,1)2k + binom(n,0)

n(n-1) * 2k^2+n * 2k + 1

(2k+1)*2k * 2k^2+(2k+1) * 2k + 1

4k^3*(2k+1) +(2k+1) * 2k + 1

8k^4* + 4k^3 +(2k+1) * 2k + 1

4k^3 + 4k^2+ 2k + 1

4k^2(k+1) +2k+1

É claro que k(k+1) é par, logo 4k^2(k+1) é 0 módulo 8

E isso nos deixa com 2k+1=n


> --
> Esta mensagem foi verificada pelo sistema de antivírus e
> acredita-se estar livre de perigo.

-- 
Esta mensagem foi verificada pelo sistema de antiv�rus e
 acredita-se estar livre de perigo.


=
Instru��es para entrar na lista, sair da lista e usar a lista em
http://www.mat.puc-rio.br/~obmlistas/obm-l.html
=


[obm-l] Re: [obm-l] Re: [obm-l] Re: [obm-l] Re: [obm-l] Re: [obm-l] Teoria dos números

2018-03-26 Por tôpico Pedro José
Bom dia!
Agora estou contente. Posso alardear que pelo menos matei um problema da
IMO.

(s-1)(t-1)(u-1) | ust-1 1=2 e só atende quando k(s,t,u) é
inteiro.
Fixando-se duas váriaveis  k é monótona decrescente para a outra; assim
kmax(s) = k(s,s+1,s+2)= (s(s+1)(s+2)-1)/(s-1)s(s+1)>=2, então
s(s+1)(s+2)/s(s-1)(s+1)>2; s < 4.

fazendo um estudo de paridade: se uma das variáveis for par as outras duas
também serão e k será ímpar. Se uma das variáveis for ímpar, todas serão
ímpares e k poderá ser tanto ímpar quanto par.

u   s   v  k
P  P   P I
III  -

s=2. k>=3 Para kmax (2,t) = k(2,t,t+1) = (2t(t+1)-1)/t(t-1)>=3 então:
2t(t+1)/t(t-1) >3 : t < 5, pela paridade t=4 e kmax(2,4) = 47/15, só serve
k = 3.

s=2, t=4 e k=3 temos v=8. (2,4,8)

s=3 k>=2 Para kmax (3,t) = k(3,t,t+1) = (3t(t+1)-1)/2t(t-1)>=2 então:
3t(t+1)/2t(t-1) >2 : t < 7, pela paridade t=5 e kmax(3,5) = 13/6, só serve
k = 2.

s=3, t= 5 e k=2 temos v= 15. (3,5,15)

Só atendem: (2,4,8) e (3,5,15)

Achei curioso que em ambas soluções, u=st.

Saudações,
PJMS



Em 26 de março de 2018 09:44, Matheus Secco 
escreveu:

> De fato, trata-se do problema 1 da IMO 1992.
>
> Abs,
>
> Matheus Secco
>
> Em Seg, 26 de mar de 2018 09:24, Claudio Buffara <
> claudio.buff...@gmail.com> escreveu:
>
>> Muito fácil pra ser de IMO...
>>
>> 2018-03-26 6:58 GMT-03:00 Anderson Torres :
>>
>>> Este não é o problema de alguma IMO não? Eu lembro de ter resolvido,
>>> quase igual à solução oficial: substituir s,t,u por a+1,b+1,c+1 e
>>> calcular os possiveis valores de
>>> 1/a+1/b+1/c + 1/ab+1/ac+1/bc usando desigualdades - para daí limitar
>>> os valores de a,b,c.
>>>
>>> Em 23 de março de 2018 17:01, Claudio Buffara
>>>  escreveu:
>>> > Enfim, nesse meio tempo acho que resolvi o problema...
>>> >
>>> > Devemos achar inteiros s, t, u, com 1 < s < t < u e tais que:
>>> > (stu -1)/((s-1)(t-1)(u-1)) = k  (k inteiro positivo)
>>> >
>>> > Após diversas aplicações do truque (método?) de somar e subtrair a
>>> mesma
>>> > coisa, chegamos a:
>>> > stu - 1 =  (s-1)(t-1)(u-1) + (s-1)(t-1) + (s-1)(u-1) + (t-1)(u-1) +
>>> (s-1) +
>>> > (t-1) + (u-1)
>>> >
>>> > Dividindo isso por (s-1)(t-1)(u-1), obtemos:
>>> > 1 + 1/(u-1) + 1/(t-1) + 1/(s-1) + 1/((t-1)(u-1)) + 1/((s-1)(u-1)) +
>>> > 1/((s-1)(t-1)) = k ==>
>>> >
>>> > 1/(u-1) + 1/(t-1) + 1/(s-1) + 1/((t-1)(u-1)) + 1/((s-1)(u-1)) +
>>> > 1/((s-1)(t-1)) = k-1
>>> >
>>> > Agora a ideia é achar cotas para s e para k.
>>> >
>>> > 1 < s < t < u ==> s >= 2, t >= 3 e u >= 4 ==> o lado esquerdo é menor
>>> ou
>>> > igual que:
>>> > 1/3 + 1/2 + 1 + 1/6 + 1/3 + 1/2 = 2+5/6
>>> >
>>> > Ou seja, como o lado esquerdo é inteiro (e positivo), só poderá ser
>>> igual a
>>> > 1 ou a 2 ==> k = 2 ou k = 3.
>>> >
>>> > Se s >= 4, então t >= 5 e u >= 6, e o lado esquerdo será, no máximo,
>>> igual
>>> > a:
>>> > 1/5 + 1/4 + 1/3 + 1/20 + 1/15 + 1/12 < 1.
>>> >
>>> > Logo, devemos ter s = 2 ou s = 3.
>>> >
>>> > s = 2 ==>
>>> > 1/(u-1) + 1/(t-1) + 1 + 1/((t-1)(u-1)) + 1/(u-1) + 1/(t-1) = k-1 ==>
>>> > 2/(t-1) + 2/(u-1) + 1/((t-1)(u-1)) = k-2 ==>
>>> > Como k-2 deve ser inteiro positivo, k só pode ser 3 e, portanto:
>>> > 2/(t-1) + 2/(u-1) + 1/((t-1)(u-1)) = 1 ==>
>>> > (2 + 1/(t-1))/(u-1) = 1 - 2/(t-1) ==>
>>> > u = 1 + (2t - 1)/(t - 3) = 3 + 5/(t-3) ==>
>>> > t = 4 e u = 8   ou   t = 8 e u = 4 (não serve pois t deve ser menor do
>>> que
>>> > u)
>>> >
>>> > s = 3 ==>
>>> > 1/(u-1) + 1/(t-1) + 1/2 + 1/((t-1)(u-1)) + 1/(2(u-1)) + 1/(2(t-1)) =
>>> k-1 ==>
>>> > (3/2)/(u-1) + (3/2)/(t-1) + 1/((t-1)(u-1)) = k - 3/2 ==>
>>> > 3/(u-1) + 3/(t-1) + 2/((t-1)(u-1)) = 2k - 3 ==>
>>> > (3 + 2/(t-1))/(u-1) = 2k - 3t/(t-1) ==>
>>> > (3t - 1)/(u-1) = 2k(t-1) - 3t ==>
>>> > u = 1 + (3t - 1)/((2k-3)t - 2k)
>>> >
>>> > k = 2 ==> u = 1 + (3t-1)/(t-4) = 4 + 11/(t-4) ==> t = 5 e u = 15
>>> >
>>> > k = 3 ==> u = 1 + (3t-1)/(3t-6) = 2 + 5/(3t-6) ==> XXX
>>> >
>>> > As únicas soluções são:
>>> > (2,4,8) e (3,5,15)
>>> >
>>> > []s,
>>> > Claudio.
>>> >
>>> > 2018-03-23 15:38 GMT-03:00 Pedro José :
>>> >>
>>> >> Boa tarde!
>>> >>
>>> >> Aproveitando que deu o que falar o problema postado pelo Douglas, tem
>>> um
>>> >> que achei mais interessante.
>>> >>
>>> >> (s-1)(t-1).(u-1) | stu -1, com s, t, u inteiros  e 1 > >>
>>> >> Saudações,
>>> >> Pedro
>>> >>
>>> >> --
>>> >> Esta mensagem foi verificada pelo sistema de antivírus e
>>> >> acredita-se estar livre de perigo.
>>> >
>>> >
>>> >
>>> > --
>>> > Esta mensagem foi verificada pelo sistema de antivírus e
>>> > acredita-se estar livre de perigo.
>>>
>>> --
>>> Esta mensagem foi verificada pelo sistema de antivírus e
>>>  acredita-se estar livre de perigo.
>>>
>>>
>>> 
>>> =
>>> Instru�ões para entrar na lista, sair da lista e usar a lista em
>>> http://www.mat.puc-rio.br/~obmlistas/obm-l.html
>>> 

[obm-l] Re: [obm-l] Re: [obm-l] Re: [obm-l] Re: [obm-l] Teoria dos números

2018-03-26 Por tôpico Matheus Secco
De fato, trata-se do problema 1 da IMO 1992.

Abs,

Matheus Secco

Em Seg, 26 de mar de 2018 09:24, Claudio Buffara 
escreveu:

> Muito fácil pra ser de IMO...
>
> 2018-03-26 6:58 GMT-03:00 Anderson Torres :
>
>> Este não é o problema de alguma IMO não? Eu lembro de ter resolvido,
>> quase igual à solução oficial: substituir s,t,u por a+1,b+1,c+1 e
>> calcular os possiveis valores de
>> 1/a+1/b+1/c + 1/ab+1/ac+1/bc usando desigualdades - para daí limitar
>> os valores de a,b,c.
>>
>> Em 23 de março de 2018 17:01, Claudio Buffara
>>  escreveu:
>> > Enfim, nesse meio tempo acho que resolvi o problema...
>> >
>> > Devemos achar inteiros s, t, u, com 1 < s < t < u e tais que:
>> > (stu -1)/((s-1)(t-1)(u-1)) = k  (k inteiro positivo)
>> >
>> > Após diversas aplicações do truque (método?) de somar e subtrair a mesma
>> > coisa, chegamos a:
>> > stu - 1 =  (s-1)(t-1)(u-1) + (s-1)(t-1) + (s-1)(u-1) + (t-1)(u-1) +
>> (s-1) +
>> > (t-1) + (u-1)
>> >
>> > Dividindo isso por (s-1)(t-1)(u-1), obtemos:
>> > 1 + 1/(u-1) + 1/(t-1) + 1/(s-1) + 1/((t-1)(u-1)) + 1/((s-1)(u-1)) +
>> > 1/((s-1)(t-1)) = k ==>
>> >
>> > 1/(u-1) + 1/(t-1) + 1/(s-1) + 1/((t-1)(u-1)) + 1/((s-1)(u-1)) +
>> > 1/((s-1)(t-1)) = k-1
>> >
>> > Agora a ideia é achar cotas para s e para k.
>> >
>> > 1 < s < t < u ==> s >= 2, t >= 3 e u >= 4 ==> o lado esquerdo é menor ou
>> > igual que:
>> > 1/3 + 1/2 + 1 + 1/6 + 1/3 + 1/2 = 2+5/6
>> >
>> > Ou seja, como o lado esquerdo é inteiro (e positivo), só poderá ser
>> igual a
>> > 1 ou a 2 ==> k = 2 ou k = 3.
>> >
>> > Se s >= 4, então t >= 5 e u >= 6, e o lado esquerdo será, no máximo,
>> igual
>> > a:
>> > 1/5 + 1/4 + 1/3 + 1/20 + 1/15 + 1/12 < 1.
>> >
>> > Logo, devemos ter s = 2 ou s = 3.
>> >
>> > s = 2 ==>
>> > 1/(u-1) + 1/(t-1) + 1 + 1/((t-1)(u-1)) + 1/(u-1) + 1/(t-1) = k-1 ==>
>> > 2/(t-1) + 2/(u-1) + 1/((t-1)(u-1)) = k-2 ==>
>> > Como k-2 deve ser inteiro positivo, k só pode ser 3 e, portanto:
>> > 2/(t-1) + 2/(u-1) + 1/((t-1)(u-1)) = 1 ==>
>> > (2 + 1/(t-1))/(u-1) = 1 - 2/(t-1) ==>
>> > u = 1 + (2t - 1)/(t - 3) = 3 + 5/(t-3) ==>
>> > t = 4 e u = 8   ou   t = 8 e u = 4 (não serve pois t deve ser menor do
>> que
>> > u)
>> >
>> > s = 3 ==>
>> > 1/(u-1) + 1/(t-1) + 1/2 + 1/((t-1)(u-1)) + 1/(2(u-1)) + 1/(2(t-1)) =
>> k-1 ==>
>> > (3/2)/(u-1) + (3/2)/(t-1) + 1/((t-1)(u-1)) = k - 3/2 ==>
>> > 3/(u-1) + 3/(t-1) + 2/((t-1)(u-1)) = 2k - 3 ==>
>> > (3 + 2/(t-1))/(u-1) = 2k - 3t/(t-1) ==>
>> > (3t - 1)/(u-1) = 2k(t-1) - 3t ==>
>> > u = 1 + (3t - 1)/((2k-3)t - 2k)
>> >
>> > k = 2 ==> u = 1 + (3t-1)/(t-4) = 4 + 11/(t-4) ==> t = 5 e u = 15
>> >
>> > k = 3 ==> u = 1 + (3t-1)/(3t-6) = 2 + 5/(3t-6) ==> XXX
>> >
>> > As únicas soluções são:
>> > (2,4,8) e (3,5,15)
>> >
>> > []s,
>> > Claudio.
>> >
>> > 2018-03-23 15:38 GMT-03:00 Pedro José :
>> >>
>> >> Boa tarde!
>> >>
>> >> Aproveitando que deu o que falar o problema postado pelo Douglas, tem
>> um
>> >> que achei mais interessante.
>> >>
>> >> (s-1)(t-1).(u-1) | stu -1, com s, t, u inteiros  e 1  >>
>> >> Saudações,
>> >> Pedro
>> >>
>> >> --
>> >> Esta mensagem foi verificada pelo sistema de antivírus e
>> >> acredita-se estar livre de perigo.
>> >
>> >
>> >
>> > --
>> > Esta mensagem foi verificada pelo sistema de antivírus e
>> > acredita-se estar livre de perigo.
>>
>> --
>> Esta mensagem foi verificada pelo sistema de antivírus e
>>  acredita-se estar livre de perigo.
>>
>>
>> =
>> Instru�ões para entrar na lista, sair da lista e usar a lista em
>> http://www.mat.puc-rio.br/~obmlistas/obm-l.html
>> =
>>
>
>
> --
> Esta mensagem foi verificada pelo sistema de antivírus e
> acredita-se estar livre de perigo.

-- 
Esta mensagem foi verificada pelo sistema de antiv�rus e
 acredita-se estar livre de perigo.



[obm-l] Re: [obm-l] Re: [obm-l] Re: [obm-l] Teoria dos números

2018-03-26 Por tôpico Claudio Buffara
Muito fácil pra ser de IMO...

2018-03-26 6:58 GMT-03:00 Anderson Torres :

> Este não é o problema de alguma IMO não? Eu lembro de ter resolvido,
> quase igual à solução oficial: substituir s,t,u por a+1,b+1,c+1 e
> calcular os possiveis valores de
> 1/a+1/b+1/c + 1/ab+1/ac+1/bc usando desigualdades - para daí limitar
> os valores de a,b,c.
>
> Em 23 de março de 2018 17:01, Claudio Buffara
>  escreveu:
> > Enfim, nesse meio tempo acho que resolvi o problema...
> >
> > Devemos achar inteiros s, t, u, com 1 < s < t < u e tais que:
> > (stu -1)/((s-1)(t-1)(u-1)) = k  (k inteiro positivo)
> >
> > Após diversas aplicações do truque (método?) de somar e subtrair a mesma
> > coisa, chegamos a:
> > stu - 1 =  (s-1)(t-1)(u-1) + (s-1)(t-1) + (s-1)(u-1) + (t-1)(u-1) +
> (s-1) +
> > (t-1) + (u-1)
> >
> > Dividindo isso por (s-1)(t-1)(u-1), obtemos:
> > 1 + 1/(u-1) + 1/(t-1) + 1/(s-1) + 1/((t-1)(u-1)) + 1/((s-1)(u-1)) +
> > 1/((s-1)(t-1)) = k ==>
> >
> > 1/(u-1) + 1/(t-1) + 1/(s-1) + 1/((t-1)(u-1)) + 1/((s-1)(u-1)) +
> > 1/((s-1)(t-1)) = k-1
> >
> > Agora a ideia é achar cotas para s e para k.
> >
> > 1 < s < t < u ==> s >= 2, t >= 3 e u >= 4 ==> o lado esquerdo é menor ou
> > igual que:
> > 1/3 + 1/2 + 1 + 1/6 + 1/3 + 1/2 = 2+5/6
> >
> > Ou seja, como o lado esquerdo é inteiro (e positivo), só poderá ser
> igual a
> > 1 ou a 2 ==> k = 2 ou k = 3.
> >
> > Se s >= 4, então t >= 5 e u >= 6, e o lado esquerdo será, no máximo,
> igual
> > a:
> > 1/5 + 1/4 + 1/3 + 1/20 + 1/15 + 1/12 < 1.
> >
> > Logo, devemos ter s = 2 ou s = 3.
> >
> > s = 2 ==>
> > 1/(u-1) + 1/(t-1) + 1 + 1/((t-1)(u-1)) + 1/(u-1) + 1/(t-1) = k-1 ==>
> > 2/(t-1) + 2/(u-1) + 1/((t-1)(u-1)) = k-2 ==>
> > Como k-2 deve ser inteiro positivo, k só pode ser 3 e, portanto:
> > 2/(t-1) + 2/(u-1) + 1/((t-1)(u-1)) = 1 ==>
> > (2 + 1/(t-1))/(u-1) = 1 - 2/(t-1) ==>
> > u = 1 + (2t - 1)/(t - 3) = 3 + 5/(t-3) ==>
> > t = 4 e u = 8   ou   t = 8 e u = 4 (não serve pois t deve ser menor do
> que
> > u)
> >
> > s = 3 ==>
> > 1/(u-1) + 1/(t-1) + 1/2 + 1/((t-1)(u-1)) + 1/(2(u-1)) + 1/(2(t-1)) = k-1
> ==>
> > (3/2)/(u-1) + (3/2)/(t-1) + 1/((t-1)(u-1)) = k - 3/2 ==>
> > 3/(u-1) + 3/(t-1) + 2/((t-1)(u-1)) = 2k - 3 ==>
> > (3 + 2/(t-1))/(u-1) = 2k - 3t/(t-1) ==>
> > (3t - 1)/(u-1) = 2k(t-1) - 3t ==>
> > u = 1 + (3t - 1)/((2k-3)t - 2k)
> >
> > k = 2 ==> u = 1 + (3t-1)/(t-4) = 4 + 11/(t-4) ==> t = 5 e u = 15
> >
> > k = 3 ==> u = 1 + (3t-1)/(3t-6) = 2 + 5/(3t-6) ==> XXX
> >
> > As únicas soluções são:
> > (2,4,8) e (3,5,15)
> >
> > []s,
> > Claudio.
> >
> > 2018-03-23 15:38 GMT-03:00 Pedro José :
> >>
> >> Boa tarde!
> >>
> >> Aproveitando que deu o que falar o problema postado pelo Douglas, tem um
> >> que achei mais interessante.
> >>
> >> (s-1)(t-1).(u-1) | stu -1, com s, t, u inteiros  e 1 >
> >> Saudações,
> >> Pedro
> >>
> >> --
> >> Esta mensagem foi verificada pelo sistema de antivírus e
> >> acredita-se estar livre de perigo.
> >
> >
> >
> > --
> > Esta mensagem foi verificada pelo sistema de antivírus e
> > acredita-se estar livre de perigo.
>
> --
> Esta mensagem foi verificada pelo sistema de antivírus e
>  acredita-se estar livre de perigo.
>
>
> =
> Instru�ões para entrar na lista, sair da lista e usar a lista em
> http://www.mat.puc-rio.br/~obmlistas/obm-l.html
> =
>

-- 
Esta mensagem foi verificada pelo sistema de antiv�rus e
 acredita-se estar livre de perigo.



[obm-l] Re: [obm-l] Teorema fundamental da álgebra

2018-03-26 Por tôpico Anderson Torres
Em 24 de março de 2018 20:13, Carlos P.  escreveu:
> Boa noite!
>
> Estou estudando análise complexa e gostaria de alguns esclarecimentos sobre
> o TFA.
>
> 1) Na prova baseada no teorema de Liouville, as únicas propriedades de
> polinômios de grau >= 1 utilizadas é que são funções inteiras tais que lim z
> ---> oo p(z) = oo. Logo, o teorema aplica-se igualmente a qualquer inteira f
> tal que lim z ---> oo f(z) = oo, certo? Não está restrito a polinômios.
>
> 2) Alguém conhece uma prova do TFA que, além de mostrar a existência de
> raízes, mostre que há exatamente n raízes, contando suas ordens? Me
> informaram que há uma

Mas isso é imediato, não? Se você demonstra que todo polinômio tem ao
menos uma raiz complexa, basta fatorar!

Por exemplo, sabendo que x^2+1 tem uma raiz, é só aplicar o mesmo para
(x^2+1)/(x-raiz)

Não sei qual o interesse que haveria em algo maior que isso, um
teorema que mostre de uma vez todas as raízes...

>
> Muito obrigado
>
> Carlos
> --
> Esta mensagem foi verificada pelo sistema de antivírus e
> acredita-se estar livre de perigo.

-- 
Esta mensagem foi verificada pelo sistema de antiv�rus e
 acredita-se estar livre de perigo.


=
Instru��es para entrar na lista, sair da lista e usar a lista em
http://www.mat.puc-rio.br/~obmlistas/obm-l.html
=


[obm-l] Re: [obm-l] Re: [obm-l] Teoria dos números

2018-03-26 Por tôpico Anderson Torres
Este não é o problema de alguma IMO não? Eu lembro de ter resolvido,
quase igual à solução oficial: substituir s,t,u por a+1,b+1,c+1 e
calcular os possiveis valores de
1/a+1/b+1/c + 1/ab+1/ac+1/bc usando desigualdades - para daí limitar
os valores de a,b,c.

Em 23 de março de 2018 17:01, Claudio Buffara
 escreveu:
> Enfim, nesse meio tempo acho que resolvi o problema...
>
> Devemos achar inteiros s, t, u, com 1 < s < t < u e tais que:
> (stu -1)/((s-1)(t-1)(u-1)) = k  (k inteiro positivo)
>
> Após diversas aplicações do truque (método?) de somar e subtrair a mesma
> coisa, chegamos a:
> stu - 1 =  (s-1)(t-1)(u-1) + (s-1)(t-1) + (s-1)(u-1) + (t-1)(u-1) + (s-1) +
> (t-1) + (u-1)
>
> Dividindo isso por (s-1)(t-1)(u-1), obtemos:
> 1 + 1/(u-1) + 1/(t-1) + 1/(s-1) + 1/((t-1)(u-1)) + 1/((s-1)(u-1)) +
> 1/((s-1)(t-1)) = k ==>
>
> 1/(u-1) + 1/(t-1) + 1/(s-1) + 1/((t-1)(u-1)) + 1/((s-1)(u-1)) +
> 1/((s-1)(t-1)) = k-1
>
> Agora a ideia é achar cotas para s e para k.
>
> 1 < s < t < u ==> s >= 2, t >= 3 e u >= 4 ==> o lado esquerdo é menor ou
> igual que:
> 1/3 + 1/2 + 1 + 1/6 + 1/3 + 1/2 = 2+5/6
>
> Ou seja, como o lado esquerdo é inteiro (e positivo), só poderá ser igual a
> 1 ou a 2 ==> k = 2 ou k = 3.
>
> Se s >= 4, então t >= 5 e u >= 6, e o lado esquerdo será, no máximo, igual
> a:
> 1/5 + 1/4 + 1/3 + 1/20 + 1/15 + 1/12 < 1.
>
> Logo, devemos ter s = 2 ou s = 3.
>
> s = 2 ==>
> 1/(u-1) + 1/(t-1) + 1 + 1/((t-1)(u-1)) + 1/(u-1) + 1/(t-1) = k-1 ==>
> 2/(t-1) + 2/(u-1) + 1/((t-1)(u-1)) = k-2 ==>
> Como k-2 deve ser inteiro positivo, k só pode ser 3 e, portanto:
> 2/(t-1) + 2/(u-1) + 1/((t-1)(u-1)) = 1 ==>
> (2 + 1/(t-1))/(u-1) = 1 - 2/(t-1) ==>
> u = 1 + (2t - 1)/(t - 3) = 3 + 5/(t-3) ==>
> t = 4 e u = 8   ou   t = 8 e u = 4 (não serve pois t deve ser menor do que
> u)
>
> s = 3 ==>
> 1/(u-1) + 1/(t-1) + 1/2 + 1/((t-1)(u-1)) + 1/(2(u-1)) + 1/(2(t-1)) = k-1 ==>
> (3/2)/(u-1) + (3/2)/(t-1) + 1/((t-1)(u-1)) = k - 3/2 ==>
> 3/(u-1) + 3/(t-1) + 2/((t-1)(u-1)) = 2k - 3 ==>
> (3 + 2/(t-1))/(u-1) = 2k - 3t/(t-1) ==>
> (3t - 1)/(u-1) = 2k(t-1) - 3t ==>
> u = 1 + (3t - 1)/((2k-3)t - 2k)
>
> k = 2 ==> u = 1 + (3t-1)/(t-4) = 4 + 11/(t-4) ==> t = 5 e u = 15
>
> k = 3 ==> u = 1 + (3t-1)/(3t-6) = 2 + 5/(3t-6) ==> XXX
>
> As únicas soluções são:
> (2,4,8) e (3,5,15)
>
> []s,
> Claudio.
>
> 2018-03-23 15:38 GMT-03:00 Pedro José :
>>
>> Boa tarde!
>>
>> Aproveitando que deu o que falar o problema postado pelo Douglas, tem um
>> que achei mais interessante.
>>
>> (s-1)(t-1).(u-1) | stu -1, com s, t, u inteiros  e 1 
>> Saudações,
>> Pedro
>>
>> --
>> Esta mensagem foi verificada pelo sistema de antivírus e
>> acredita-se estar livre de perigo.
>
>
>
> --
> Esta mensagem foi verificada pelo sistema de antivírus e
> acredita-se estar livre de perigo.

-- 
Esta mensagem foi verificada pelo sistema de antiv�rus e
 acredita-se estar livre de perigo.


=
Instru��es para entrar na lista, sair da lista e usar a lista em
http://www.mat.puc-rio.br/~obmlistas/obm-l.html
=


[obm-l] Re: [obm-l] Re: [obm-l] Re: [obm-l] Re: [obm-l] Re: [obm-l] Teoria dos números

2018-03-26 Por tôpico Anderson Torres
Em 23 de março de 2018 10:35, Claudio Buffara
 escreveu:
> Na verdade os meus questionamentos surgiram por causa do meu interesse em
> ensino de matemática.
>
> Por exemplo, produtos notáveis e fatorações são notoriamente mal ensinados,
> pelo menos nos livros didáticos de 8o e 9o ano que eu examinei.

Eu acho que uma motivação mais geométrica pode ser bastante útil para
muitos produtos notáveis.

Por exemplo, a diferença de quadrados é bem facilmente explicada de
forma geométrica:
um quadrado com um quadradinho a menos no canto pode ser quebrado em
dois trapézios que formam
um retângulo.
O quadrado da soma é mais fácil ainda.

Por outro lado, eu não penso que minha solução foi a mais mágica de
todas, apenas era desconhecida.
Sempre que noto uma expressão simétrica, eu penso em como escrevê-la
em função ou dos
polinômios simétricos elementares ou da soma de potências (x^k+y^k+z^k).

> Nenhum menciona que:
> a) as generalizações de (x+y)^2 = x^2 + 2xy + y^2  e  x^2 - y^2 = (x-y)(x+y)
> para expoentes maiores levam ao teorema do binômio (erroneamente chamado de
> binômio de Newton - nota histórica: Newton generalizou o teorema para
> expoentes racionais) e à fórmula da soma dos termos de uma PG;
> b) (x+y)^2 = x^2 + 2xy + y^2 é a base para a ideia de se completar
> quadrados, a qual, por sua vez, não só resulta na fórmula para as raízes de
> uma equação quadrática, mas também na elucidação das propriedades da função
> quadrática;
> c) o uso inteligente da expansão de (x+y)^3 leva à formula das raízes de uma
> equação cúbica.

Essa eu não conhecia. Ainda penso que as formas mais naturais de lidar
com a cúbica são o Método Gugu-Euler
(tentar uma solução da forma x=raizcúbica(y1)+raizcúbica(y2)) ou usar
a fatoração de x^3+y^3+z^3-3xyz.

Mas o cubo da soma, per se? Isso me parece mágico demais.

(Acho que até imagino o que seja: identificar o termo constante com a
soma de cubos e o termo "linear" com o triplo do produto...)

>
> ***
>
> Há tempos, o Hermann, participante desta lista, postou uma dúvida sobre
> produtos notáveis e pediu dicas de livros com exercícios sobre produtos
> notáveis e fatoração.
> Eu tenho duas sugestões, ambas em inglês:
> - Algebra, de I.M.Gelfand e A.Shen - Birkhäuser (este faz as generalizações
> que eu mencionei acima)
> - A Problem Book in Algebra, de V.A. Krechmar - Mir Publishers (pros
> entusiastas)
> Ambos estão disponíveis na Amazon.
>
> ***
>
> Anos atrás eu gostava de soluções "mágicas", obtidas por meio de alguma
> sacada brilhante que eu jamais conseguiria ter.
> Após me deparar com várias destas soluções, me ocorreu que elas talvez
> tivessem um efeito perverso na motivação dos estudantes de matemática, pois
> passavam a impressão de que é preciso ser um gênio para dominar a matéria.
> Daí o meu interesse em saber como vocês obtiveram certas fatorações.
> Entendo que trabalho braçal, experiência, alguma lógica e um pouco de
> otimismo são, para a maioria de nós, as únicas formas de progredir na
> resolução de um problema como o que deu origem a este thread.
>
> Dito isso (e posso estar enganado) nem o Pedro José e nem mais ninguém
> explicou de onde veio a conjectura (correta) de que:
> z = -(x+y)/2 é solução de (x + y)(y + z)(z + x)/2 + (x + y + z)3 =  – xyz
>
> []s,
> Claudio.
>
>
>
> 2018-03-23 6:20 GMT-03:00 Anderson Torres :
>>
>> Em 21 de março de 2018 09:47, Claudio Buffara
>>  escreveu:
>> > Como você passou de:
>> > 4abc + (a+b+c)^3 + (-a+b+c)(a-b+c)(a+b-c) = 1
>> >
>> > Para:
>> > 4(a+b+c)(ab+ac+bc) - 4abc = 1
>>
>> It's kind of magic. Eu simplesmente abri tudo com vontade e notei
>> certas repetições
>> que sempre aparecem em certas fatorações; ou melhor dizendo, estava
>> pensando em
>> escrever tudo em termos dos famigerados polinômios simétricos e cheguei
>> nisso.
>>
>> Sempre que vejo algo como (a^2b+ab^2), já escrevo ab(a+b) e tento
>> procurar um abc
>> para isso resultar em ab(a+b+c).
>>
>> Mas não avancei daí. Penso que dá para fatorar ainda mais...
>>
>> >
>> > ???
>> >
>> > []s,
>> > Claudio.
>> >
>> >
>> > 2018-03-20 23:14 GMT-03:00 Anderson Torres
>> > :
>> >>
>> >> Em 13 de março de 2018 20:19, Douglas Oliveira de Lima
>> >>  escreveu:
>> >> > Essa achei legal e estou postando.
>> >> >
>> >> > Resolva nos inteiros a seguinte equação:  (x + y)(y + z)(z + x)/2 +
>> >> > (x +
>> >> > y +
>> >> > z)3 = 1 – xyz .
>> >> >
>> >>
>> >> Substituição mágica: x=-a+b+c, y=a-b+c, z=a+b-c. Com isso, x+y=2c,
>> >> x+y+z=a+b+c e
>> >>
>> >> 4abc + (a+b+c)^3 + (-a+b+c)(a-b+c)(a+b-c) = 1
>> >>
>> >> Usando polinômios simétricos,
>> >>
>> >> 4(a+b+c)(ab+ac+bc) - 4abc = 1
>> >>
>> >> Agora estou confuso...
>> >>
>> >> > Abraço do
>> >> > Douglas Oliveira
>> >> >
>> >> > --
>> >> > Esta mensagem foi verificada pelo sistema de antivírus e
>> >> > acredita-se estar livre de perigo.
>> >>
>> >> --
>> >> Esta mensagem foi